Đến nội dung

vuvanquya1nct nội dung

Có 299 mục bởi vuvanquya1nct (Tìm giới hạn từ 24-05-2020)



Sắp theo                Sắp xếp  

#407130 Topic về Phương trình và hệ phương trình không mẫu mực

Đã gửi bởi vuvanquya1nct on 22-03-2013 - 22:26 trong Phương trình, hệ phương trình và bất phương trình

Góp thêm một bài :
Giải PT :
$\sqrt[3]{1 - x} + \sqrt{x + 2} = 1$

phương trình đã cho tương đương:
$(\sqrt[3]{1-x}+1)+(\sqrt{x+2}-2)=0$$\Leftrightarrow \frac{2-x}{(\sqrt[3]{1-x})^2-\sqrt[3]{1-x}+1}-\frac{2-x}{\sqrt{2+x}+2}=0$
đặt nhân tử chung (2-x)




#475870 $\sqrt[3]{2x+4}-\sqrt[3]{2x-1}=\sqrt[3]{5}$

Đã gửi bởi vuvanquya1nct on 06-01-2014 - 22:43 trong Phương trình, hệ phương trình và bất phương trình

BÀI 7:
Giải phương trình: $\left( {x + 3} \right)\sqrt {2{x^2} + 1} = {x^2} + x + 3$
___

PT $\Leftrightarrow 2(x+3)\sqrt{2x^2+1}=2x^2+2x+6$

$(2x^2+1)-2(x+3)\sqrt{2x^2+1}+2x+5=0$ (*)

Ta có $\Delta '=(x+2)^2$

Nên (*)$\Leftrightarrow \begin{bmatrix} \sqrt{2x^2+1}=5+2x & \\ \sqrt{2x^2+1}=1 & \end{bmatrix}$

$\Leftrightarrow \begin{bmatrix} x=0 & \\ x=-5+\sqrt{13} & \end{bmatrix}$




#487016 $\begin{cases}x^3-3x^2+2x-5=y \\ y^3+3y^2+2y-4=z\end...

Đã gửi bởi vuvanquya1nct on 15-03-2014 - 20:20 trong Phương trình, hệ phương trình và bất phương trình

Bạn nào giúp mình làm câu này với :'(

Có lẽ theo mình thế $y=2x^3-3x$ vào PT thứ 2:

$x^2-1=\sqrt{x^2-4x+5}(2x^3-4x-3)$

Nhưng mà đến đây cũng chẳng đơn giản...Cho thời gian nhé!!!!!

:>




#479366 $\begin{cases}x^3-3x^2+2x-5=y \\ y^3+3y^2+2y-4=z\end...

Đã gửi bởi vuvanquya1nct on 27-01-2014 - 13:13 trong Phương trình, hệ phương trình và bất phương trình

giải hệ phương trình :

$\left\{\begin{matrix} 2x+\frac{1}{x+y}=3\\ 4xy+4(x^2+y^2)+3:(x+y)^2=7 \end{matrix}\right.$

p/s dài quá latex bị lỗi, ko dùng phân số đc, thông cảm

Hệ $\Leftrightarrow \left\{\begin{matrix} (x+y)+(x-y)+\frac{1}{x+y}=3 & \\ 3(x+y)^2+(x-y)^2+\frac{3}{(x+y)^2}=7 & \end{matrix}\right.$

Đặt $\left\{\begin{matrix} a=x+y & \\ b=x-y & \end{matrix}\right.$

Ta được hê:

$\left\{\begin{matrix} a+b+\frac{1}{a}=3 & \\4(a^2+\frac{1}{a^2})+b^2=7 & \end{matrix}\right.$

$\Leftrightarrow \left\{\begin{matrix} (a+\frac{1}{a})+b=3 & \\4(a+\frac{1}{a})^2+b^2=15 & \end{matrix}\right.$

Giải hệ trên là gần Xong




#486055 Đề Thi HSG Tỉnh Lớp 9 Tỉnh Hà Tĩnh Năm 2013-2014

Đã gửi bởi vuvanquya1nct on 06-03-2014 - 19:52 trong Tài liệu - Đề thi

Câu $1$ ,

a, Giải phương trình $2\sqrt{2x-1}=x^2+1.$

b, Giải hệ phương trình $$\begin{cases}3x^3+xy^2=2y \\ y^3+x^2y=-2x. \end{cases}$$

Câu b nhân chéo để mà ra PT đẳng cấp suy ra mối quan hệ x và y

Câu a

ĐK...

PT$\Leftrightarrow (x+1)^2=(\sqrt{2x-1}+1)^2$

Đến đây ok




#459425 Đề thi HSG Tp Hà Nội lớp 12 năm 2013-2014

Đã gửi bởi vuvanquya1nct on 23-10-2013 - 15:09 trong Thi HSG cấp Tỉnh, Thành phố. Olympic 30-4. Đề thi và kiểm tra đội tuyển các cấp.

Câu 1b.

Gọi A(a,y(a))   B(b,y(b))  và C(c,y(c))     (a,b,c khác 0 và đôi một khác nhau)

Điêì kiện để A , B , C thẳng hàng là  $\overrightarrow{AB}=k.\overrightarrow{AC}$$\Leftrightarrow \frac{x_A-x_B}{y_A-y_B}=\frac{x_A-x_C}{y_A-y_C} a=b=c$

Viết phương Trình tiếp tuyến tai A.B,C

 Tìm Giao điểm A' , B' , C'   (theo a, b, c)

Tổng hoành độ của ba điểm A' , B' , C' =-2(a+b+c)=0

Suy ra A' , B' , C' thẳng hàng




#478459 Chứng minh rằng : $\frac{3}{xy+yz+xz}+\fr...

Đã gửi bởi vuvanquya1nct on 22-01-2014 - 15:00 trong Đại số

Cho $x,y,z\geq 0$ thỏa mãn $x+y+z=1$. Chứng minh rằng : 

$\frac{3}{xy+yz+xz}+\frac{2}{x^2+y^2+z^2}\geq14$

$VP=2(\frac{1}{x^2+y^2+z^2}+\frac{1}{2(xy+yz+zx)})+\frac{2}{xy+yz+zx}$

$\geq 8+\frac{2}{xy+yz+zx}$

BĐT quy về CM  8+\frac{2}{xy+yz+zx}\geq 14$

$\Leftrightarrow$$\frac{2}{xy+yz+zx}\geq 6$

$\Leftrightarrow \frac{1}{xy+yz+zx}\geq 3$

$\Leftrightarrow$$3(xy+yz+zx)\leq 1 \Leftrightarrow 3(xy+yz+zx)\leq(x+y+z)^2=1$

Dấu bằng không xảy ra !!!!!




#459424 Đề thi HSG Tp Hà Nội lớp 12 năm 2013-2014

Đã gửi bởi vuvanquya1nct on 23-10-2013 - 15:09 trong Thi HSG cấp Tỉnh, Thành phố. Olympic 30-4. Đề thi và kiểm tra đội tuyển các cấp.

Câu 1b.

Gọi A(a,y(a))   B(b,y(b))  và C(c,y(c))     (a,b,c khác 0 và đôi một khác nhau)

Điêì kiện để A , B , C thẳng hàng là  $\overrightarrow{AB}=k.\overrightarrow{AC}$$\Leftrightarrow \frac{x_A-x_B}{y_A-y_B}=\frac{x_A-x_C}{y_A-y_C} a=b=c$

Viết phương Trình tiếp tuyến tai A.B,C

 Tìm Giao điểm A' , B' , C'   (theo a, b, c)

Tổng hoành độ của ba điểm A' , B' , C' =-2(a+b+c)=0

Suy ra A' , B' , C' thẳng hàng




#475853 Ai có đề Casio cấp quốc qia không

Đã gửi bởi vuvanquya1nct on 06-01-2014 - 22:10 trong Giải toán bằng máy tính bỏ túi

Cho e hỏi là khi nào thi Casio cấp QG nhỉ?????




#475961 Ai có đề Casio cấp quốc qia không

Đã gửi bởi vuvanquya1nct on 07-01-2014 - 15:03 trong Giải toán bằng máy tính bỏ túi

Khoảng giữa tháng 3 , đầu tháng 4 bạn à

Bạn có đề casio toán THPT Kg???




#472074 Lập công thức truy hồi: $U_{n}={(\frac{3+\...

Đã gửi bởi vuvanquya1nct on 21-12-2013 - 18:30 trong Giải toán bằng máy tính bỏ túi

bạn làm thế không sai nhưng thầy mình bảo làm vậy chưa được chặt chẽ!

Có khi nó chỉ đúng cho n=1,2,3,4,5.

Còn chưa chắc đã đúng với mọi n




#466250 Giải phương trình: 1) $x^{3}+1=3\sqrt[3]{3x-1}...

Đã gửi bởi vuvanquya1nct on 23-11-2013 - 17:31 trong Phương trình - hệ phương trình - bất phương trình

Bài 2: PT $< = > (x^2-4x+2)^2=x+2<= > (x^2-4x)^2+4(x^2-4x)+4=x+2< = > x^4-8x^3+16x^2+4x^2-16x+4=x+2< = > x^4-8x^3+20x^2-17x+2=0$

đến đây giải pt bậc 4 sao nữa nhỉ




#466196 Giải phương trình: 1) $x^{3}+1=3\sqrt[3]{3x-1}...

Đã gửi bởi vuvanquya1nct on 23-11-2013 - 12:18 trong Phương trình - hệ phương trình - bất phương trình

Giải phương trình:

1) $x^{3}+1=3\sqrt[3]{3x-1}$

 

đặt $y=\sqrt[3]{3x-1}\Rightarrow y^3+1=3x$

Được hệ $\left\{\begin{matrix} x^3+1=3y & \\ y^3+1=3x & \end{matrix}\right.$

=> Hệ đối xứng




#472116 Đề ra kì này số 438

Đã gửi bởi vuvanquya1nct on 21-12-2013 - 21:17 trong Toán học & Tuổi trẻ

Bài 1: Giải phương trình :$8\sqrt{2x^2-x^4}+16\sqrt{x^2+2x^4}=1$



Bài 2: Cho $x-y\geq 0,x+y\geq 0,\sqrt{(\frac{x+y}{2})^3}+\sqrt{(\frac{x-y}{2})^3}=27$

Tìm Min của x

bạn 

Daicagiangho1998

đề co đúng không vậy??

Bạn up ảnh lên được không?

P/s: Cảm ơn




#468361 $\left\{\begin{matrix}x^{3}+4y=y...

Đã gửi bởi vuvanquya1nct on 02-12-2013 - 15:32 trong Phương trình - hệ phương trình - bất phương trình

$\left\{\begin{matrix}x^{3}+4y=y^{3}+4x\\ x^{4}+2y^{2}=1\end{matrix}\right.$ 

 xét hàm số $f(t)=t^3+4t$

$f'(t)=3t^2+4>0$ với mọi t thuộc R suy ra hàm f(t) đồng biến ,kết hợp phương trình đầu

suy ra x=y thế vào pt 2 xong




#468511 $\left\{\begin{matrix}x^{3}+4y=y...

Đã gửi bởi vuvanquya1nct on 03-12-2013 - 12:08 trong Phương trình - hệ phương trình - bất phương trình

Bạn lạm dụng hàm sô quá nhỉ??

Nhưng bạn xét hàm nhầm rồi!!! Phải là $f(t)=t^3-4t$ Lấy điều kiện phương trình (2) thì hàm số này nghịch biến. OK

thank. x y nhiều hoa mắt.già rồi !!!!!!1

:)




#400419 $\lim_{x\rightarrow \infty}\left ( \s...

Đã gửi bởi vuvanquya1nct on 27-02-2013 - 17:57 trong Dãy số - Giới hạn

Có thể áp dụng dạng hằng đẳng thức để triệt tiêu dạng vô định đó:$A-B=(\sqrt{A}-\sqrt{B})(\sqrt{A}+\sqrt{B})$
Ta luôn có :$\sqrt{x-2}-\sqrt{x+2}=\frac{-4}{sqrt{x-2}+\sqrt{x+2}$
Trên tử bậc o
Dưới mẫu bậc 1
Chia tử và nẫu cho x, =>lim=0



#486023 $\left\{\begin{matrix} x_{1}=2 & \\ x_{n}=...

Đã gửi bởi vuvanquya1nct on 06-03-2014 - 11:54 trong Dãy số - Giới hạn

1.$\left\{\begin{matrix} x_{1}=2 & \\ x_{n}=\frac{x_{n-1}^2+2}{2x_{n}} & \end{matrix}\right.$

2.$\left\{\begin{matrix} u{1}=2 & \\ u_{n+1}=\frac{u_{n}^2}{2u_{n}-1} & \end{matrix}\right.$




#483400 $x^{2}+4x+7=(x+4)\sqrt{x^{2}+7}$

Đã gửi bởi vuvanquya1nct on 16-02-2014 - 10:03 trong Phương trình, hệ phương trình và bất phương trình

muốn bỏ giá trị tuyệt đối phải xét trường hợp của nó bạn ạ!

Dieu kien la ve phai lon hon hoac bang khong.Tuc la

$VP=\frac{1}{2}(2x^3+x^2+2x+1)=(x+\frac{1}{2})(x^2+1)\geq 0$

$\Leftrightarrow x+\frac{1}{2}\geq 0$

Do do khoi can tri tuyet doi xet truong hop




#483268 $x^{2}+4x+7=(x+4)\sqrt{x^{2}+7}$

Đã gửi bởi vuvanquya1nct on 15-02-2014 - 19:12 trong Phương trình, hệ phương trình và bất phương trình

Bài 2

ĐK...

$PT\Leftrightarrow (X^2+7)-(x+4)\sqrt{x^2+7}+4x=0$  (*)

Ta có $\Delta =(x-4)^2$ nên (*) $\Leftrightarrow \begin{bmatrix} \sqrt{x^2+7}=-x & \\ \sqrt{x^2+7}=-4 & \end{bmatrix}$

Đến đây OK

Bài này tớ đã nhầm cái đoạn tô đỏ,Sửa lại là $(*)\Leftrightarrow \begin{bmatrix} \sqrt{x^2+7}=4 & \\ \sqrt{x^2+7}=x & \end{bmatrix}$

Và đến đây mới OK

Còn bài 1

ĐK....

PT$\Leftrightarrow \sqrt{x^2-\frac{1}{4}+\left | x+\frac{1}{2} \right |}=(2x+1)(x^2+1)$

$\Leftrightarrow x+\frac{1}{2}=(2x+1)(x^2+1)$

Đến đây thì quá dễ




#473693 Cho các số thực x, y, z thỏa mãn $\left\{\begin...

Đã gửi bởi vuvanquya1nct on 29-12-2013 - 15:03 trong Đại số

Bạn giải tiếp mấy phương trình này đi. Nhân tiện chỉ cho mình cách giải phương trình đẳng cấp với.

Ví dụ như phương trình:$x^2+3xy-4y^2=0$ là 1 phương trình đẳng cấp 

Xét y=0 (hoặc x=0 )có phải là nghiệm không,nếu không ta chia hai vế cho y^2 (hoặc x^2):

$(\frac{x}{y})^2+3(\frac{x}{y})-4=0$

Và đây là phương trình bậc 2 theo ẩn là x/y

PT đó có nghiệm $\begin{bmatrix} \frac{x}{y}=1 & \\ \frac{x}{y}=-4 & \end{bmatrix}$




#471044 $\cot \frac{A}{2}.\cot\frac...

Đã gửi bởi vuvanquya1nct on 15-12-2013 - 10:02 trong Công thức lượng giác, hàm số lượng giác

Không phải,ở đây nè :

 

Nếu là công thức hay gì đó về lượng giác anh cho em và mọi người biết nhé  !! ( Em là học sinh lớp 10)

Chõ đó nhầm phải là  $cos\frac{A-C}{2}.sin\frac{A+C}{2}=2.cos\frac{A+c}{2}.sin\frac{A+C}{2}$

(nhân cả hai vế cho $sin\frac{A+C}{2}$ ý mà)

Đã sửa lại ở bài viết




#473654 Cho các số thực x, y, z thỏa mãn $\left\{\begin...

Đã gửi bởi vuvanquya1nct on 29-12-2013 - 12:05 trong Đại số

 

Bài 2: Giải phương trình:

a) $5\sqrt{x^{3}+1}=2(x^{2}+2)$

b) $2(x^{2}+2x+3)=5\sqrt{x^{3}+3x^{2}+3x+2}$

 

ĐK......

a)PT$\Leftrightarrow 5\sqrt{(x+1)(x^2-x+1)}=2(x^2-x+1)+2(x+1)$

Và đây chính là pt đẳng cấp.

b)PT$\Leftrightarrow 5\sqrt{(x+2)(x^2+x+1)}=2(x+2)+2(x^2+x+1)$

Đây cũng là phương trình đẳng cấp.




#475842 Phương pháp dùng tính đơn điệu của hàm số để giải phương trình, hệ...

Đã gửi bởi vuvanquya1nct on 06-01-2014 - 22:03 trong Phương trình - Hệ phương trình - Bất phương trình

Bài 10:

Đặt $y=\sqrt[3]{7x^2+9x-4}$

Ta có hệ $\left\{\begin{matrix} y^3=7x^2+9x-4 & \\ y=x^3-4x^2-5x+6 & \end{matrix}\right.$

Cộng vế theo vế ta được $y^3+y=(x+1)^3+(x+1)$  (*)

Xét hàm $f(t)=t^3+t$ đông biến trên R với mọi t.nên từ (*) suy ra y=x+1

Quy về giải PT $x+1=\sqrt[3]{7x^2+9x-4}$

$\Leftrightarrow \begin{bmatrix} x=5 & \\ x=\frac{-1\pm \sqrt{5}}{2} & \end{bmatrix}$




#478327 Giải phương trình : $2x^2-6x-1=\sqrt{4x+5}$

Đã gửi bởi vuvanquya1nct on 21-01-2014 - 12:25 trong Phương trình, hệ phương trình và bất phương trình



 

Giải phương trình :

  • $2x^2-6x-1=\sqrt{4x+5}$

 

  • $2x^2+4x=\sqrt{\frac{x+3}{2}}$

 

  • $4004x-2001=\left ( \frac{8x^3+2001}{2002} \right )^3$

 

Bài cuối số TO:

PT $\Leftrightarrow$ $\sqrt[3]{4004x-2001}=\frac{8x^3+2001}{2002}$

Đặt $\sqrt[3]{4004x-2001}=2y$

Ta chuyển về hệ đối xứng này $\left\{\begin{matrix} 4004x-2001=8y^3 & \\ 4004y-2001=8x^3 & \end{matrix}\right.$

Trừ vế theo vế là Xong !!!